LSAT and Law School Admissions Forum

Get expert LSAT preparation and law school admissions advice from PowerScore Test Preparation.

 Administrator
PowerScore Staff
  • PowerScore Staff
  • Posts: 8917
  • Joined: Feb 02, 2011
|
#47135
Complete Question Explanation
(The complete setup for this game can be found here: lsat/viewtopic.php?t=2909)

The correct answer choice is (C)

As discussed during the analysis of the third rule, Y, G, and O are each used in one or two windows. Thus, none of them can be used in all three windows, and so answer choices (A), (B), and (E) can be eliminated.

Answer choice (D) can be eliminated because the second rule specifies that R is used in exactly two windows.

Answer choice (C) is thus proven correct by process of elimination. Additionally, the hypothetical from question #10 proves this answer choice correct as well.

Get the most out of your LSAT Prep Plus subscription.

Analyze and track your performance with our Testing and Analytics Package.